Finding Sup and Inf in Real Analysis: x^2 - 5x + 6 < 0 and x^2 + 1 = 0

  • Thread starter Thread starter converting1
  • Start date Start date
Click For Summary
The discussion focuses on finding the supremum and infimum of specific sets in real analysis. For the set defined by x^2 - 5x + 6 < 0, the infimum is 2 and the supremum is 3. The second set, interpreted as the range of the function x^2 - 5x + 6, has an infimum of -1/4 and no supremum, as it is unbounded above. The final set, x^2 + 1 = 0, has no real solutions, indicating that both the supremum and infimum do not exist. Overall, the reasoning provided for each set is confirmed to be correct.
converting1
Messages
65
Reaction score
0
find the sup and inf of the following sets:

{ x | x^2 - 5x + 6 &lt; 0 }

I got the inf and sup to be 2 and 3 respectively

{ x^2 - 5x + 6 | x \in ℝ}
here I was rather confused what this is saying. I'm assuming it's taking about the graph x^2 - 5x + 6 and assumed this was between [-1/4, ∞] so inf = -1/4 and sup does not exist as it is not bounded from above.

{x | x^2 + 1 = 0 }
as I'm in a real analysis class, there isn't a real number such that x^2 + 1 = 0, so inf and sup do not exist

could anyone check my answers and if my reasoning is correct, especially for the second one please

I don't understand why the curly brackets are not showing, but there should be curly brackets around all above in tex
 
Physics news on Phys.org
converting1 said:
find the sup and inf of the following sets:

\{ x | x^2 - 5x + 6 &lt; 0 \}

I got the inf and sup to be 2 and 3 respectively

Looks good.

{ x^2 - 5x + 6 | x \in ℝ}
here I was rather confused what this is saying. I'm assuming it's taking about the graph x^2 - 5x + 6 and assumed this was between [-1/4, ∞] so inf = -1/4 and sup does not exist as it is not bounded from above.

It's talking about the range, so yes, that looks right too.

{x | x^2 + 1 = 0 }
as I'm in a real analysis class, there isn't a real number such that x^2 + 1 = 0, so inf and sup do not exist

could anyone check my answers and if my reasoning is correct, especially for the second one please

I don't understand why the curly brackets are not showing, but there should be curly brackets around all above in tex

The curly brackets have a special use in TeX, so to display then you use \{ and \} as I did editing your first set.
 
LCKurtz said:
Looks good.



It's talking about the range, so yes, that looks right too.



The curly brackets have a special use in TeX, so to display then you use \{ and \} as I did editing your first set.

thank you for a fast reply,

is the last one correct too as you did not comment on that?
 
converting1 said:
thank you for a fast reply,

is the last one correct too as you did not comment on that?

I would say so as long as your text doesn't have some special convention for empty sets.
 
  • Like
Likes 1 person
Question: A clock's minute hand has length 4 and its hour hand has length 3. What is the distance between the tips at the moment when it is increasing most rapidly?(Putnam Exam Question) Answer: Making assumption that both the hands moves at constant angular velocities, the answer is ## \sqrt{7} .## But don't you think this assumption is somewhat doubtful and wrong?

Similar threads

Replies
11
Views
2K
Replies
6
Views
3K
  • · Replies 2 ·
Replies
2
Views
2K
Replies
2
Views
2K
  • · Replies 3 ·
Replies
3
Views
3K
  • · Replies 2 ·
Replies
2
Views
5K
Replies
4
Views
4K
  • · Replies 15 ·
Replies
15
Views
2K
  • · Replies 2 ·
Replies
2
Views
2K
  • · Replies 11 ·
Replies
11
Views
2K